0 Daumen
602 Aufrufe

Zeigen Sie, dass die folgende Reihe divergiert.

\( \sum \limits_{n=1}^{\infty} \frac{3}{n}= \)

Avatar von

1 Antwort

0 Daumen
Du kannst die Drei ausklammern:

$$\sum_{n=1}^{\infty} \frac{3}{n} = 3 \cdot \sum_{n=1}^{\infty} \frac{1}{n}$$

Die letzte Summe ist bekanntlich die divergente harmonische Reihe. Daher divergiert die Reihe insgesamt.
Avatar von 4,3 k

Ein anderes Problem?

Stell deine Frage

Willkommen bei der Mathelounge! Stell deine Frage einfach und kostenlos

x
Made by a lovely community